The Loophole in Logical Reasoning - Question Stems & Types
The answer to which one of the following questions is most relevant to evaluating the conclusion drawn above?
Evaluate
The answer to which one of the following would be the most helpful in determining whether William's argument could be logically defended against Marianna's objection?
Evaluate
Which one of the following would it be most helpful to know in order to judge whether what the scientist subsequently learned calls into question the hypothesis?
Evaluate
Which one of the following would it be most relevant to investigate in evaluating the conclusion of the argument?
Evaluate
Which one of the following most accurately expresses the main conclusion of the argument?
Conclusion
Which one of the following most accurately expresses the overall conclusion drawn in the argument?
Conclusion
If the statements above are true, each of the following could be true EXCEPT:
Contradiction
If the statements above are true, then on the basis of them which one of the following cannot be true?
Contradiction
The information above, if accurate, can best be used as evidence against which one of the following hypotheses?
Contradiction
Max and Shirin disagree over whether:
Controversy
The exchange between JT and Carolina most strongly supports the view that they disagree as to
Controversy
The statements above provide the most support for holding that Sandra would disagree with Yul about which one of the following statements?
Controversy
Which one of the following is the point at issue between Aubry and Tai?
Controversy
Which one of the following, if true, is the strongest counter Laila can make to Timothy's objection?
Counter
Which one of the following, if true, would provide Sina with the strongest response to Gabriela's response?
Counter
The advertisement's reasoning is most vulnerable to criticism on the grounds that it fails to consider whether:
Loophole Flaw
The reasoning in the researcher's argument is questionable in that the argument overlooks the possibility that:
Loophole Flaw
The scholar's reasoning is flawed because the scholar presumes without giving sufficient justification that:
Loophole Flaw
Of the following, which one most accurately describes Tom's strategy of argumentation?
Method
The relationship of Denise's response to Malcolm's argument is that Denise's response:
Method
Wendell responds to Domenick's argument by:
Method
Which one of the following most accurately describes how the argument proceeds?
Method
The information above provides the most support for which one of the following statements?
Most Strongly Supported
The statements above, if true, most strongly support which one of the following?
Most Strongly Supported
Which one of the following is most strongly supported by the information above?
Most Strongly Supported
The argument's conclusion can be properly drawn if which one of the following is assumed?
Sufficient Assumption
The conclusion drawn above follows logically if which one of the following is assumed?
Sufficient Assumption
Which one of the following is an assumption that would allow the conclusion above to be properly drawn?
Sufficient Assumption
The argument assumes which one of the following?
Necessary Assumption
The argument depends on assuming that:
Necessary Assumption
Which one of the following is an assumption on which the argument relies?
Necessary Assumption
Which one of the following is an assumption required by the argument?
Necessary Assumption
The flawed reasoning in which one of the following arguments most closely resembles the flawed reasoning in the argument above?
Parallel Flaw
Which one of the following arguments exhibits flawed reasoning that is most parallel to that in the argument above?
Parallel Flaw
Which one of the following is most appropriate as an analogy demonstrating that the reasoning in the argument above is flawed?
Parallel Flaw
The pattern of reasoning in the argument above is most similar to that in which one of the following?
Parallel Reasoning
The pattern of the reasoning in the argument above is most closely paralleled in which one of the following?
Parallel Reasoning
The reasoning in which one of the following is most similar to the reasoning above?
Parallel Reasoning
The reasoning above conforms most closely to which one of the following propositions?
Principle Conform
Which one of the following conforms most closely to the principle illustrated above?
Principle Conform
Which one of the following propositions is best illustrated by the passage?
Principle Conform
Which one of the following propositions is best illustrated by the situation described in the passage?
Principle Conform
Which one of the following, if true, most helps to strengthen the argument?
Strengthen
Which one of the following, if true, most strengthens the argument?
Strengthen
If Retta and Han are both sincere in what they say, then it can be properly concluded that they agree that:
Agreement
On the basis of their statements, Jim and Pam are committed to agreeing about which one of the following?
Agreement
The reference to the complaint of several centuries ago that powerful memory and extemporaneous eloquence were being destroyed plays which one of the following roles in the argument?
Argument Part
The statement that the law should require explicit safety labels on toys serves which one of the following functions in the consumer advocate's argument?
Argument Part
The reasoning in the argument is most vulnerable to criticism on the grounds that the argument:
Classic Flaw
Which one of the following is a questionable technique employed by the producer in responding to the critic?
Classic Flaw
Which one of the following most accurately describes a flaw in the reasoning of the argument?
Classic Flaw
The conclusion of the argument is strongly supported if which one of the following completes the argument?
Fill In
Which one of the following most logically completes the argument?
Fill In
If the statement above is true, which one of the following must also be true?
Inference
If the statements above are true, which one of the following is an inference that can be properly drawn on the basis of them?
Inference
Which one of the following can be properly inferred from the statements above?
Inference
Which one of the following can properly be concluded from the information given above?
Inference
Which one of the following statements follows logically from the statements above?
Inference
Which one of the following, if true, does most to explain the surprising result?
Resolution
Which one of the following, if true, helps to resolve the apparent conflict described above?
Resolution
Which one of the following, if true, most helps to resolve the apparent discrepancy in the information above?
Resolution
Which one of the following, if true, most helps to resolve the apparent paradox?
Resolution
Which one of the following principles, if valid, most helps to justify the reasoning in the argument?
Strengthen
Which one of the following principles, if valid, most strongly supports the reasoning above?
Strengthen
Which one of the following, if true, most seriously weakens the argument above?
Weaken
Which one of the following, if true, most tends to undermine the argument?
Weaken